Difficult Math Question #32 - Sets

This topic has expert replies
Master | Next Rank: 500 Posts
Posts: 354
Joined: Tue Jun 27, 2006 9:20 pm
Thanked: 11 times
Followed by:5 members

Difficult Math Question #32 - Sets

by 800guy » Mon Oct 23, 2006 2:27 pm
A set of numbers has the property that for any number t in the set, t + 2 is in the set. If –1 is in the set, which of the following must also be in the set?
I. –3 II. 1 III. 5
A. I only
B. II only
C. I and II only
D. II and III only
E. I, II, and III

Senior | Next Rank: 100 Posts
Posts: 87
Joined: Fri Jun 09, 2006 2:47 am
Thanked: 2 times

by rajs.kumar » Tue Oct 24, 2006 2:55 am
D it is. {-1, 1, 3, ....., +inf}

Newbie | Next Rank: 10 Posts
Posts: 5
Joined: Mon Oct 23, 2006 8:57 am

by girikoratagere » Tue Oct 24, 2006 7:31 am
Is it C
?

User avatar
Legendary Member
Posts: 1275
Joined: Thu Sep 21, 2006 11:13 pm
Location: Arabian Sea
Thanked: 125 times
Followed by:2 members

by ajith » Tue Oct 24, 2006 8:32 pm
D it is ..

since -1 is there 1 has to be there
similarly since 1 is there 3 has to be there
so is 5

-3 may or may not be there...

User avatar
Senior | Next Rank: 100 Posts
Posts: 57
Joined: Wed Jul 12, 2006 3:13 pm
Followed by:4 members

by limits660 » Wed Oct 25, 2006 4:27 am
I think its E

All of them are there
-
Jeff Sacco
www.jeffsacco.ca

Master | Next Rank: 500 Posts
Posts: 354
Joined: Tue Jun 27, 2006 9:20 pm
Thanked: 11 times
Followed by:5 members

OA

by 800guy » Wed Oct 25, 2006 3:28 pm
here's the OA:

Series property: t => t+2. (Note: for any given number N, ONLY N + 2 is compulsory. N - 2 is not a necessity as N could be the first term...this can be used as a trap.)
Given: -1 belongs to the series. => 1 => 3 =>5. DOES NOT imply -3.
Hence, II and III (D).

Junior | Next Rank: 30 Posts
Posts: 11
Joined: Sun May 31, 2020 9:47 pm

Re: Difficult Math Question #32 - Sets

by jasminekaur280 » Wed Jun 03, 2020 12:41 am
A set of numbers has the property that for any number t in the set, t + 2 is in the set. If –1 is in the set, which of the following must also be in the set?
I. –3 II. 1 III. 5
A. I only
B. II only
C. I and II only
D. II and III only
E. I, II, and III